• Shuffle
    Toggle On
    Toggle Off
  • Alphabetize
    Toggle On
    Toggle Off
  • Front First
    Toggle On
    Toggle Off
  • Both Sides
    Toggle On
    Toggle Off
  • Read
    Toggle On
    Toggle Off
Reading...
Front

Card Range To Study

through

image

Play button

image

Play button

image

Progress

1/117

Click to flip

Use LEFT and RIGHT arrow keys to navigate between flashcards;

Use UP and DOWN arrow keys to flip the card;

H to show hint;

A reads text to speech;

117 Cards in this Set

  • Front
  • Back
“If the statements above are true...”
MUST BE TRUE

Fact Test
Right answers:
1) Paraphrase - reworded portion of stimulus.
2) Combination of 2+ stimulus

Wrong answers:
1) Any information that does not appear either directly or as a combination of items in the stimulus will be incorrect.
2) Exagerated answers - pay attention to modifiers (some vs. most).
3) Reversal of logic (A->B becomes B->A) or modifiers (Many people have some vs. Some people have many)
4) Something that could be or likely to be true.
5) Opposite

Stimulus -> Answer Choices "under suspicion
“The statements above, if true...”
MUST BE TRUE

Fact Test
Right answers:
1) Paraphrase - reworded portion of stimulus.
2) Combination of 2+ stimulus

Wrong answers:
1) Any information that does not appear either directly or as a combination of items in the stimulus will be incorrect.
2) Exagerated answers - pay attention to modifiers (some vs. most).
3) Reversal of logic (A->B becomes B->A) or modifiers (Many people have some vs. Some people have many)
4) Something that could be or likely to be true.
5) Opposite

Stimulus -> Answer Choices "under suspicion
“If the information above is correct...”
MUST BE TRUE

Fact Test
Right answers:
1) Paraphrase - reworded portion of stimulus.
2) Combination of 2+ stimulus

Wrong answers:
1) Any information that does not appear either directly or as a combination of items in the stimulus will be incorrect.
2) Exagerated answers - pay attention to modifiers (some vs. most).
3) Reversal of logic (A->B becomes B->A) or modifiers (Many people have some vs. Some people have many)
4) Something that could be or likely to be true.
5) Opposite

Stimulus -> Answer Choices "under suspicion
“...which one of the following must also be true?”
MUST BE TRUE

Fact Test
Right answers:
1) Paraphrase - reworded portion of stimulus.
2) Combination of 2+ stimulus

Wrong answers:
1) Any information that does not appear either directly or as a combination of items in the stimulus will be incorrect.
2) Exagerated answers - pay attention to modifiers (some vs. most).
3) Reversal of logic (A->B becomes B->A) or modifiers (Many people have some vs. Some people have many)
4) Something that could be or likely to be true.
5) Opposite

Stimulus -> Answer Choices "under suspicion
“...which one of the following conclusions can be properly
drawn on the basis of it?”
MUST BE TRUE

Fact Test
Right answers:
1) Paraphrase - reworded portion of stimulus.
2) Combination of 2+ stimulus

Wrong answers:
1) Any information that does not appear either directly or as a combination of items in the stimulus will be incorrect.
2) Exagerated answers - pay attention to modifiers (some vs. most).
3) Reversal of logic (A->B becomes B->A) or modifiers (Many people have some vs. Some people have many)
4) Something that could be or likely to be true.
5) Opposite

Stimulus -> Answer Choices "under suspicion
“...most strongly support which one of the following?”
MUST BE TRUE

Fact Test
Right answers:
1) Paraphrase - reworded portion of stimulus.
2) Combination of 2+ stimulus

Wrong answers:
1) Any information that does not appear either directly or as a combination of items in the stimulus will be incorrect.
2) Exagerated answers - pay attention to modifiers (some vs. most).
3) Reversal of logic (A->B becomes B->A) or modifiers (Many people have some vs. Some people have many)
4) Something that could be or likely to be true.
5) Opposite

Stimulus -> Answer Choices "under suspicion
“Which one of the following can be properly inferred...”
MUST BE TRUE

Fact Test
Right answers:
1) Paraphrase - reworded portion of stimulus.
2) Combination of 2+ stimulus

Wrong answers:
1) Any information that does not appear either directly or as a combination of items in the stimulus will be incorrect.
2) Exagerated answers - pay attention to modifiers (some vs. most).
3) Reversal of logic (A->B becomes B->A) or modifiers (Many people have some vs. Some people have many)
4) Something that could be or likely to be true.
5) Opposite

Stimulus -> Answer Choices "under suspicion
“If the statements above are true, which one of the following must also be true?”
MUST BE TRUE

Fact Test
Right answers:
1) Paraphrase - reworded portion of stimulus.
2) Combination of 2+ stimulus

Wrong answers:
1) Any information that does not appear either directly or as a combination of items in the stimulus will be incorrect.
2) Exagerated answers - pay attention to modifiers (some vs. most).
3) Reversal of logic (A->B becomes B->A) or modifiers (Many people have some vs. Some people have many)
4) Something that could be or likely to be true.
5) Opposite

Stimulus -> Answer Choices "under suspicion
“If the information above is correct, which one of the following conclusions can be properly drawn on the basis of it?”
MUST BE TRUE

Fact Test
Right answers:
1) Paraphrase - reworded portion of stimulus.
2) Combination of 2+ stimulus

Wrong answers:
1) Any information that does not appear either directly or as a combination of items in the stimulus will be incorrect.
2) Exagerated answers - pay attention to modifiers (some vs. most).
3) Reversal of logic (A->B becomes B->A) or modifiers (Many people have some vs. Some people have many)
4) Something that could be or likely to be true.
5) Opposite

Stimulus -> Answer Choices "under suspicion
“The statements above, if true, most strongly support which one of the following?”
MUST BE TRUE

Fact Test
Right answers:
1) Paraphrase - reworded portion of stimulus.
2) Combination of 2+ stimulus

Wrong answers:
1) Any information that does not appear either directly or as a combination of items in the stimulus will be incorrect.
2) Exagerated answers - pay attention to modifiers (some vs. most).
3) Reversal of logic (A->B becomes B->A) or modifiers (Many people have some vs. Some people have many)
4) Something that could be or likely to be true.
5) Opposite

Stimulus -> Answer Choices "under suspicion
“Which one of the following can be properly inferred from the
passage?”
MUST BE TRUE

Fact Test
Right answers:
1) Paraphrase - reworded portion of stimulus.
2) Combination of 2+ stimulus

Wrong answers:
1) Any information that does not appear either directly or as a combination of items in the stimulus will be incorrect.
2) Exagerated answers - pay attention to modifiers (some vs. most).
3) Reversal of logic (A->B becomes B->A) or modifiers (Many people have some vs. Some people have many)
4) Something that could be or likely to be true.
5) Opposite

Stimulus -> Answer Choices "under suspicion
“Which one of the following is most strongly supported by the
information above?”
MUST BE TRUE

Fact Test
Right answers:
1) Paraphrase - reworded portion of stimulus.
2) Combination of 2+ stimulus

Wrong answers:
1) Any information that does not appear either directly or as a combination of items in the stimulus will be incorrect.
2) Exagerated answers - pay attention to modifiers (some vs. most).
3) Reversal of logic (A->B becomes B->A) or modifiers (Many people have some vs. Some people have many)
4) Something that could be or likely to be true.
5) Opposite

Stimulus -> Answer Choices "under suspicion
“Which one of the following most accurately expresses the main
conclusion of the argument?”
MAIN POINT
Right answer:
1) Find the primary conclusion.
2) Look for words that introduce a conclusion.

Wrong Answers:
1) Answers that are true, but not main point.
2) Answers that repeat the premise of the argument.
2) Any information that does not appear either directly or as a combination of items in the stimulus will be incorrect.

Stimulus -> Answer Choices "under suspicion"
“Which one of the following most accurately expresses the conclusion
of the journalist’s argument?”
MAIN POINT
Right answer:
1) Find the primary conclusion.
2) Look for words that introduce a conclusion.

Wrong Answers:
1) Answers that are true, but not main point.
2) Answers that repeat the premise of the argument.
2) Any information that does not appear either directly or as a combination of items in the stimulus will be incorrect.

Stimulus -> Answer Choices "under suspicion"
“Which one of the following most accurately restates the main point of
the passage?”
MAIN POINT
Right answer:
1) Find the primary conclusion.
2) Look for words that introduce a conclusion.

Wrong Answers:
1) Answers that are true, but not main point.
2) Answers that repeat the premise of the argument.
2) Any information that does not appear either directly or as a combination of items in the stimulus will be incorrect.

Stimulus -> Answer Choices "under suspicion"
“The main point of the argument is that”
MAIN POINT
Right answer:
1) Find the primary conclusion.
2) Look for words that introduce a conclusion.

Wrong Answers:
1) Answers that are true, but not main point.
2) Answers that repeat the premise of the argument.
2) Any information that does not appear either directly or as a combination of items in the stimulus will be incorrect.

Stimulus -> Answer Choices "under suspicion"
“Therefore, __________.”
MAIN POINT FIB
Right answer:
1) Find the primary conclusion.
2) Look for words that introduce a conclusion.

Wrong Answers:
1) Answers that are true, but not main point.
2) Answers that repeat the premise of the argument.
2) Any information that does not appear either directly or as a combination of items in the stimulus will be incorrect.

Stimulus -> Answer Choices "under suspicion"
“Hence, in the new century, the stability of a nation’s cultural identity
will likely __________.”
MAIN POINT FIB
Right answer:
1) Find the primary conclusion.
2) Look for words that introduce a conclusion.

Wrong Answers:
1) Answers that are true, but not main point.
2) Answers that repeat the premise of the argument.
2) Any information that does not appear either directly or as a combination of items in the stimulus will be incorrect.

Stimulus -> Answer Choices "under suspicion"
“Thus, in many cases, by criminals’ characterization of their
situations, __________.”
MAIN POINT FIB
Right answer:
1) Find the primary conclusion.
2) Look for words that introduce a conclusion.

Wrong Answers:
1) Answers that are true, but not main point.
2) Answers that repeat the premise of the argument.
2) Any information that does not appear either directly or as a combination of items in the stimulus will be incorrect.

Stimulus -> Answer Choices "under suspicion"
“Which one of the following statements is most strongly
supported by the information above?”
Question Type: Must Be True
“The information above provides the LEAST support for
which one of the following?”
Question Type: Must Be True X
The four incorrect answer
choices Must Be True; the correct answer choice is not necessarily true.
“Which one of the following most logically completes the
argument?”
Question Type: Main Point-FITB
“The educators’ reasoning provides grounds for accepting
which one of the following statements?”
Question Type: Must Be True
“Which one of the following most accurately expresses the
argument’s conclusion?”
Main Point
“Which one of the following can be inferred from the
passage above?”
Must Be True
“inferred” means must be
true.
“Which one of the following, if true, most seriously weakens the
argument?”
Weaken- ask you to attack or undermine the author's argument

OFTEN attacks the NECESSARY condition by showing it doesn't need to occur for the sufficient to occur.

Family #3
1) The information in the stimulus is suspect. There are often reasoning errors present, and you will further weaken the argument in some way.

2) The answer choices are acceptable as given, even if they include "new" information. Your task is to determine which answer choice best attacks the argument in the stimulus.
3. Focus on the conclusion
Yield strong Prephrases
Recognize and Attack
1. Incomplete Information.
2. Improper Comparison.
3. Qualified Conclusion.

Stimulus <-/- Answer Choices
Use answer to hurt the argument
“Which one of the following, if true, most undermines the researcher’s
argument?”
Weaken- ask you to attack or undermine the author's argument

OFTEN attacks the NECESSARY condition by showing it doesn't need to occur for the sufficient to occur.

Family #3
1) The information in the stimulus is suspect. There are often reasoning errors present, and you will further weaken the argument in some way.

2) The answer choices are acceptable as given, even if they include "new" information. Your task is to determine which answer choice best attacks the argument in the stimulus.
3. Focus on the conclusion
Yield strong Prephrases
Recognize and Attack
1. Incomplete Information.
2. Improper Comparison.
3. Qualified Conclusion.

Stimulus <-/- Answer Choices
Use answer to hurt the argument
“Which one of the following, if shown to be a realistic possibility,
would undermine the argument?”
Weaken- ask you to attack or undermine the author's argument

OFTEN attacks the NECESSARY condition by showing it doesn't need to occur for the sufficient to occur.

Family #3
1) The information in the stimulus is suspect. There are often reasoning errors present, and you will further weaken the argument in some way.

2) The answer choices are acceptable as given, even if they include "new" information. Your task is to determine which answer choice best attacks the argument in the stimulus.
3. Focus on the conclusion
Yield strong Prephrases
Recognize and Attack
1. Incomplete Information.
2. Improper Comparison.
3. Qualified Conclusion.

Stimulus <-/- Answer Choices
Use answer to hurt the argument
“Which one of the following, if true, would most call into question the
analysts’ explanation of the price increase?”
Weaken- ask you to attack or undermine the author's argument

OFTEN attacks the NECESSARY condition by showing it doesn't need to occur for the sufficient to occur.

Family #3
1) The information in the stimulus is suspect. There are often reasoning errors present, and you will further weaken the argument in some way.

2) The answer choices are acceptable as given, even if they include "new" information. Your task is to determine which answer choice best attacks the argument in the stimulus.
3. Focus on the conclusion
Yield strong Prephrases
Recognize and Attack
1. Incomplete Information.
2. Improper Comparison.
3. Qualified Conclusion.

Stimulus <-/- Answer Choices
Use answer to hurt the argument
“Which one of the following, if true, could be used by Cora to counter
Bernard’s rejection of her explanation?”
Weaken- ask you to attack or undermine the author's argument

OFTEN attacks the NECESSARY condition by showing it doesn't need to occur for the sufficient to occur.

Family #3
1) The information in the stimulus is suspect. There are often reasoning errors present, and you will further weaken the argument in some way.

2) The answer choices are acceptable as given, even if they include "new" information. Your task is to determine which answer choice best attacks the argument in the stimulus.
3. Focus on the conclusion
Yield strong Prephrases
Recognize and Attack
1. Incomplete Information.
2. Improper Comparison.
3. Qualified Conclusion.

Stimulus <-/- Answer Choices
Use answer to hurt the argument
“Which one of the following, if true, is the strongest logical counter
parent P can make to parent Q’s objection?”
Weaken- ask you to attack or undermine the author's argument

OFTEN attacks the NECESSARY condition by showing it doesn't need to occur for the sufficient to occur.

Family #3
1) The information in the stimulus is suspect. There are often reasoning errors present, and you will further weaken the argument in some way.

2) The answer choices are acceptable as given, even if they include "new" information. Your task is to determine which answer choice best attacks the argument in the stimulus.
3. Focus on the conclusion
Yield strong Prephrases
Recognize and Attack
1. Incomplete Information.
2. Improper Comparison.
3. Qualified Conclusion.

Stimulus <-/- Answer Choices
Use answer to hurt the argument
“Which one of the following, if true, most calls into question the claim
above?”
Weaken- ask you to attack or undermine the author's argument

OFTEN attacks the NECESSARY condition by showing it doesn't need to occur for the sufficient to occur.

Family #3
1) The information in the stimulus is suspect. There are often reasoning errors present, and you will further weaken the argument in some way.

2) The answer choices are acceptable as given, even if they include "new" information. Your task is to determine which answer choice best attacks the argument in the stimulus.
3. Focus on the conclusion
Yield strong Prephrases
Recognize and Attack
1. Incomplete Information.
2. Improper Comparison.
3. Qualified Conclusion.

Stimulus <-/- Answer Choices
Use answer to hurt the argument
“Which one of the following, if true, most strengthens the argument?”
STRENGTHEN
ask you to select the answer choice that provides support (maybe as little as 1%) for the author's argument or strengthens it in some way.

For right answer:
1) Identify the conclusion - what you are trying to stregthen!
2) Personalize the argument
3) Look for weaknesses in the argument (gap between premise and conclusion)
4) Look for analogies or surveys (data) that needs to be stregthened.
5)Only needs to strengthen a little or a lot (1% or 100%)


Incorrect answers:
1) Opposite - weaken the argument.
2) Shell Games
3) Out of scope - unrelated to conclusion.

IF CAUSE AND EFFECT:
A. Eliminate any alternative causes for the stated effect- the author believes there is ONLY ONE cause for the effect, so eliminating another cause stregthens.
B. Show that when the cause occurs, the effect occurs - since the author believes the effect MUST follow the cause.
C. Show that when the cause doesn't occur the effect does not occur - because the cause is the ONLY thing that creates the effect.
D. Eliminating a reverse relationship - the effect created the cause
E. Show the data used to make the casual statement is accurate, or eliminate possible problems with the data.

Family #2

1) The information in the stimulus is suspect. There are often reasoning errors present, and depending on the question, you will help shore up the argument in some way.

2) The answer choices are accepted as given, even if they include "new" information. Your task is to determine which answer choice best meets the question posed in the stem.

Stimulus (under suspicion) <- Answer Choices
“Which one of the following, if true, most strongly supports the
statement above?”
STRENGTHEN
ask you to select the answer choice that provides support (maybe as little as 1%) for the author's argument or strengthens it in some way.

For right answer:
1) Identify the conclusion - what you are trying to stregthen!
2) Personalize the argument
3) Look for weaknesses in the argument (gap between premise and conclusion)
4) Look for analogies or surveys (data) that needs to be stregthened.
5)Only needs to strengthen a little or a lot (1% or 100%)


Incorrect answers:
1) Opposite - weaken the argument.
2) Shell Games
3) Out of scope - unrelated to conclusion.

IF CAUSE AND EFFECT:
A. Eliminate any alternative causes for the stated effect- the author believes there is ONLY ONE cause for the effect, so eliminating another cause stregthens.
B. Show that when the cause occurs, the effect occurs - since the author believes the effect MUST follow the cause.
C. Show that when the cause doesn't occur the effect does not occur - because the cause is the ONLY thing that creates the effect.
D. Eliminating a reverse relationship - the effect created the cause
E. Show the data used to make the casual statement is accurate, or eliminate possible problems with the data.

Family #2

1) The information in the stimulus is suspect. There are often reasoning errors present, and depending on the question, you will help shore up the argument in some way.

2) The answer choices are accepted as given, even if they include "new" information. Your task is to determine which answer choice best meets the question posed in the stem.

Stimulus (under suspicion) <- Answer Choices
“Which one of the following, if true, does most to justify the conclusion
above?”
STRENGTHEN
ask you to select the answer choice that provides support (maybe as little as 1%) for the author's argument or strengthens it in some way.

For right answer:
1) Identify the conclusion - what you are trying to stregthen!
2) Personalize the argument
3) Look for weaknesses in the argument (gap between premise and conclusion)
4) Look for analogies or surveys (data) that needs to be stregthened.
5)Only needs to strengthen a little or a lot (1% or 100%)


Incorrect answers:
1) Opposite - weaken the argument.
2) Shell Games
3) Out of scope - unrelated to conclusion.

IF CAUSE AND EFFECT:
A. Eliminate any alternative causes for the stated effect- the author believes there is ONLY ONE cause for the effect, so eliminating another cause stregthens.
B. Show that when the cause occurs, the effect occurs - since the author believes the effect MUST follow the cause.
C. Show that when the cause doesn't occur the effect does not occur - because the cause is the ONLY thing that creates the effect.
D. Eliminating a reverse relationship - the effect created the cause
E. Show the data used to make the casual statement is accurate, or eliminate possible problems with the data.

Family #2

1) The information in the stimulus is suspect. There are often reasoning errors present, and depending on the question, you will help shore up the argument in some way.

2) The answer choices are accepted as given, even if they include "new" information. Your task is to determine which answer choice best meets the question posed in the stem.

Stimulus (under suspicion) <- Answer Choices
“Each of the following, if true, supports the claim above EXCEPT:”
STRENGTHEN X
ask you to select the answer choice that provides support (maybe as little as 1%) for the author's argument or strengthens it in some way.

For right answer:
1) Identify the conclusion - what you are trying to stregthen!
2) Personalize the argument
3) Look for weaknesses in the argument (gap between premise and conclusion)
4) Look for analogies or surveys (data) that needs to be stregthened.
5)Only needs to strengthen a little or a lot (1% or 100%)


Incorrect answers:
1) Opposite - weaken the argument.
2) Shell Games
3) Out of scope - unrelated to conclusion.

IF CAUSE AND EFFECT:
A. Eliminate any alternative causes for the stated effect- the author believes there is ONLY ONE cause for the effect, so eliminating another cause stregthens.
B. Show that when the cause occurs, the effect occurs - since the author believes the effect MUST follow the cause.
C. Show that when the cause doesn't occur the effect does not occur - because the cause is the ONLY thing that creates the effect.
D. Eliminating a reverse relationship - the effect created the cause
E. Show the data used to make the casual statement is accurate, or eliminate possible problems with the data.

Family #2

1) The information in the stimulus is suspect. There are often reasoning errors present, and depending on the question, you will help shore up the argument in some way.

2) The answer choices are accepted as given, even if they include "new" information. Your task is to determine which answer choice best meets the question posed in the stem.

Stimulus (under suspicion) <- Answer Choices
“The conclusion above follows logically if which one of the following
is assumed?”
Justify the Conclusion- 100% PROVES the conclusion without a doubt.

RIGHT:
1) Any "new information" (not in premises) will appear in the correct answer

WRONG:
1) Elements common to the conclusion and at least not premise, do not appear in the correct answer.
2) Elements that appear in the premise but not in the conclusion are not normally in the correct answer.

Correct A --> Conclusion

If answer then conclusion must follow.
Justify Formula
Premises + Answer Choice = Conclusion
mechanistic approach
1. Any “new” element in the conclusion will appear in the correct answer.
2. common to the conclusion and at least 1 premise, or
to 2 premises, normally do not appear in the correct answer.
3. appear in the premises but not the conclusion normally
appear in the correct answer.
Family #2
“If which one of the following is taken as true (assumed), then the
conclusion above follows logically?”
Justify the Conclusion- 100% PROVES the conclusion without a doubt.

RIGHT:
1) Any "new information" (not in premises) will appear in the correct answer

WRONG:
1) Elements common to the conclusion and at least not premise, do not appear in the correct answer.
2) Elements that appear in the premise but not in the conclusion are not normally in the correct answer.

Correct A --> Conclusion

If answer then conclusion must follow.
Justify Formula
Premises + Answer Choice = Conclusion
mechanistic approach
1. Any “new” element in the conclusion will appear in the correct answer.
2. common to the conclusion and at least 1 premise, or
to 2 premises, normally do not appear in the correct answer.
3. appear in the premises but not the conclusion normally
appear in the correct answer.
Family #2
“allows the conclusion to be properly
drawn”
Justify the Conclusion- 100% PROVES the conclusion without a doubt.

RIGHT:
1) Any "new information" (not in premises) will appear in the correct answer

WRONG:
1) Elements common to the conclusion and at least not premise, do not appear in the correct answer.
2) Elements that appear in the premise but not in the conclusion are not normally in the correct answer.

Correct A --> Conclusion

If answer then conclusion must follow.
Justify Formula
Premises + Answer Choice = Conclusion
mechanistic approach
1. Any “new” element in the conclusion will appear in the correct answer.
2. common to the conclusion and at least 1 premise, or
to 2 premises, normally do not appear in the correct answer.
3. appear in the premises but not the conclusion normally
appear in the correct answer.
Family #2
“enables the conclusion to be properly drawn.”
Justify the Conclusion- 100% PROVES the conclusion without a doubt.

RIGHT:
1) Any "new information" (not in premises) will appear in the correct answer

WRONG:
1) Elements common to the conclusion and at least not premise, do not appear in the correct answer.
2) Elements that appear in the premise but not in the conclusion are not normally in the correct answer.

Correct A --> Conclusion

If answer then conclusion must follow.
Justify Formula
Premises + Answer Choice = Conclusion
mechanistic approach
1. Any “new” element in the conclusion will appear in the correct answer.
2. common to the conclusion and at least 1 premise, or
to 2 premises, normally do not appear in the correct answer.
3. appear in the premises but not the conclusion normally
appear in the correct answer.
Family #2
“The conclusion above follows logically if which one of the following
is assumed?”
Justify the Conclusion- 100% PROVES the conclusion without a doubt.

RIGHT:
1) Any "new information" (not in premises) will appear in the correct answer

WRONG:
1) Elements common to the conclusion and at least not premise, do not appear in the correct answer.
2) Elements that appear in the premise but not in the conclusion are not normally in the correct answer.

Correct A --> Conclusion

If answer then conclusion must follow.
Justify Formula
Premises + Answer Choice = Conclusion
mechanistic approach
1. Any “new” element in the conclusion will appear in the correct answer.
2. common to the conclusion and at least 1 premise, or
to 2 premises, normally do not appear in the correct answer.
3. appear in the premises but not the conclusion normally
appear in the correct answer.
Family #2
“Which one of the following, if assumed, would allow the conclusion to
be properly drawn?”
Justify the Conclusion- 100% PROVES the conclusion without a doubt.

RIGHT:
1) Any "new information" (not in premises) will appear in the correct answer

WRONG:
1) Elements common to the conclusion and at least not premise, do not appear in the correct answer.
2) Elements that appear in the premise but not in the conclusion are not normally in the correct answer.

Correct A --> Conclusion

If answer then conclusion must follow.
Justify Formula
Premises + Answer Choice = Conclusion
mechanistic approach
1. Any “new” element in the conclusion will appear in the correct answer.
2. common to the conclusion and at least 1 premise, or
to 2 premises, normally do not appear in the correct answer.
3. appear in the premises but not the conclusion normally
appear in the correct answer.
Family #2
“Which one of the following, if true, enables the conclusion to be
properly drawn?”
Justify the Conclusion- 100% PROVES the conclusion without a doubt.

RIGHT:
1) Any "new information" (not in premises) will appear in the correct answer

WRONG:
1) Elements common to the conclusion and at least not premise, do not appear in the correct answer.
2) Elements that appear in the premise but not in the conclusion are not normally in the correct answer.

Correct A --> Conclusion

If answer then conclusion must follow.
Justify Formula
Premises + Answer Choice = Conclusion
mechanistic approach
1. Any “new” element in the conclusion will appear in the correct answer.
2. common to the conclusion and at least 1 premise, or
to 2 premises, normally do not appear in the correct answer.
3. appear in the premises but not the conclusion normally
appear in the correct answer.
Family #2
“Which one of the following, if assumed, enables the argument’s
conclusion to be properly inferred?”
Justify the Conclusion- 100% PROVES the conclusion without a doubt.

RIGHT:
1) Any "new information" (not in premises) will appear in the correct answer

WRONG:
1) Elements common to the conclusion and at least not premise, do not appear in the correct answer.
2) Elements that appear in the premise but not in the conclusion are not normally in the correct answer.

Correct A --> Conclusion

If answer then conclusion must follow.
Justify Formula
Premises + Answer Choice = Conclusion
mechanistic approach
1. Any “new” element in the conclusion will appear in the correct answer.
2. common to the conclusion and at least 1 premise, or
to 2 premises, normally do not appear in the correct answer.
3. appear in the premises but not the conclusion normally
appear in the correct answer.
Family #2
“Which one of the following is an assumption that would serve to justify
the conclusion above?”
Justify the Conclusion- 100% PROVES the conclusion without a doubt.

RIGHT:
1) Any "new information" (not in premises) will appear in the correct answer

WRONG:
1) Elements common to the conclusion and at least not premise, do not appear in the correct answer.
2) Elements that appear in the premise but not in the conclusion are not normally in the correct answer.

Correct A --> Conclusion

If answer then conclusion must follow.
Justify Formula
Premises + Answer Choice = Conclusion
mechanistic approach
1. Any “new” element in the conclusion will appear in the correct answer.
2. common to the conclusion and at least 1 premise, or
to 2 premises, normally do not appear in the correct answer.
3. appear in the premises but not the conclusion normally
appear in the correct answer.
Family #2
“The environmentalist’s conclusion would be properly drawn if it were
true that the”
Justify the Conclusion- 100% PROVES the conclusion without a doubt.

RIGHT:
1) Any "new information" (not in premises) will appear in the correct answer

WRONG:
1) Elements common to the conclusion and at least not premise, do not appear in the correct answer.
2) Elements that appear in the premise but not in the conclusion are not normally in the correct answer.

Correct A --> Conclusion

If answer then conclusion must follow.
Justify Formula
Premises + Answer Choice = Conclusion
mechanistic approach
1. Any “new” element in the conclusion will appear in the correct answer.
2. common to the conclusion and at least 1 premise, or
to 2 premises, normally do not appear in the correct answer.
3. appear in the premises but not the conclusion normally
appear in the correct answer.
Family #2
“The conclusion above is properly drawn if which one of the following
is assumed?”
Justify the Conclusion- 100% PROVES the conclusion without a doubt.

RIGHT:
1) Any "new information" (not in premises) will appear in the correct answer

WRONG:
1) Elements common to the conclusion and at least not premise, do not appear in the correct answer.
2) Elements that appear in the premise but not in the conclusion are not normally in the correct answer.

Correct A --> Conclusion

If answer then conclusion must follow.
Justify Formula
Premises + Answer Choice = Conclusion
mechanistic approach
1. Any “new” element in the conclusion will appear in the correct answer.
2. common to the conclusion and at least 1 premise, or
to 2 premises, normally do not appear in the correct answer.
3. appear in the premises but not the conclusion normally
appear in the correct answer.
Family #2
“Which one of the following is an assumption required by the argument
above?”
Assumption
Assumption question stems typically contain the following features:
1. The stem uses the word “assumption” or “presupposition” or some
variation.
2. The stem does not use the word “if” or another sufficient condition
indicator.

Supporter Assumption: These assumptions link together new
elements in the stimulus or fill logical gaps in the
argument.
Defender Assumption: contain statements that
eliminate ideas that would
undermine the conclusion. In this sense, they
“defend” the argument by showing that a
possible avenue of attack has been eliminated
(assumed not to exist).

Assumption Negation becomes weaken
1. Logically negate the answer choices under consideration.
2. The negated answer choice that attacks the argument will be the correct
answer.
“Which one of the following is an assumption upon which the argument
depends?”
Assumption
Assumption question stems typically contain the following features:
1. The stem uses the word “assumption” or “presupposition” or some
variation.
2. The stem does not use the word “if” or another sufficient condition
indicator.

Supporter Assumption: These assumptions link together new
elements in the stimulus or fill logical gaps in the
argument.
Defender Assumption: contain statements that
eliminate ideas that would
undermine the conclusion. In this sense, they
“defend” the argument by showing that a
possible avenue of attack has been eliminated
(assumed not to exist).

Assumption Negation becomes weaken
1. Logically negate the answer choices under consideration.
2. The negated answer choice that attacks the argument will be the correct
answer.
“The argument assumes which one of the following?”
Assumption
Assumption question stems typically contain the following features:
1. The stem uses the word “assumption” or “presupposition” or some
variation.
2. The stem does not use the word “if” or another sufficient condition
indicator.

Supporter Assumption: These assumptions link together new
elements in the stimulus or fill logical gaps in the
argument.
Defender Assumption: contain statements that
eliminate ideas that would
undermine the conclusion. In this sense, they
“defend” the argument by showing that a
possible avenue of attack has been eliminated
(assumed not to exist).

Assumption Negation becomes weaken
1. Logically negate the answer choices under consideration.
2. The negated answer choice that attacks the argument will be the correct
answer.
“The conclusion in the passage above relies on which one of the
following assumptions?”
Assumption
Assumption question stems typically contain the following features:
1. The stem uses the word “assumption” or “presupposition” or some
variation.
2. The stem does not use the word “if” or another sufficient condition
indicator.

Supporter Assumption: These assumptions link together new
elements in the stimulus or fill logical gaps in the
argument.
Defender Assumption: contain statements that
eliminate ideas that would
undermine the conclusion. In this sense, they
“defend” the argument by showing that a
possible avenue of attack has been eliminated
(assumed not to exist).

Assumption Negation becomes weaken
1. Logically negate the answer choices under consideration.
2. The negated answer choice that attacks the argument will be the correct
answer.
“The position taken above presupposes which one of the following?”
Assumption
Assumption question stems typically contain the following features:
1. The stem uses the word “assumption” or “presupposition” or some
variation.
2. The stem does not use the word “if” or another sufficient condition
indicator.

Supporter Assumption: These assumptions link together new
elements in the stimulus or fill logical gaps in the
argument.
Defender Assumption: contain statements that
eliminate ideas that would
undermine the conclusion. In this sense, they
“defend” the argument by showing that a
possible avenue of attack has been eliminated
(assumed not to exist).

Assumption Negation becomes weaken
1. Logically negate the answer choices under consideration.
2. The negated answer choice that attacks the argument will be the correct
answer.
“The conclusion cited does not follow unless”
Assumption
Assumption question stems typically contain the following features:
1. The stem uses the word “assumption” or “presupposition” or some
variation.
2. The stem does not use the word “if” or another sufficient condition
indicator.

Supporter Assumption: These assumptions link together new
elements in the stimulus or fill logical gaps in the
argument.
Defender Assumption: contain statements that
eliminate ideas that would
undermine the conclusion. In this sense, they
“defend” the argument by showing that a
possible avenue of attack has been eliminated
(assumed not to exist).

Assumption Negation becomes weaken
1. Logically negate the answer choices under consideration.
2. The negated answer choice that attacks the argument will be the correct
answer.
“Which one of the following, if true, most strongly supports the statement above?”
Question Type: Strengthen
Stem drawn from the December 2003 LSAT. The presence of the phrase “most strongly supports”
indicates that this question stem is a Strengthen.
“Which one of the following is an assumption on which Patti’s argument depends?”
Question Type: Assumption
Stem drawn from the October 2003 LSAT. The presence of the word “assumption” without any sufficient
condition indicators classifies this question as an Assumption. Further, the phrase “depends upon” indicates
the answer will have a necessary quality, a hallmark of assumption questions.
“The conclusion drawn by the professor follows logically if which one of the
following is assumed?”
Question Type: Justify the Conclusion
Stem drawn from the December 2003 LSAT. The presence of the words “if” and “assumed” indicates that
this question stem is a Justify.
“Which one of the following is an assumption required by the argument?”
Question Type: Assumption
Stem drawn from the October 2003 LSAT. The key words in this stem are “required” and “assumption,”
making this an Assumption question. You must be careful when you see the word “assumption” because
that word can also be used in Justify the Conclusion questions.
“Which one of the following, if assumed, would allow the conclusion to be properly
drawn?”
Question Type: Justify the Conclusion
Stem drawn from the October 2003 LSAT. The presence of the words “if” and “assumed” indicates that
this question stem is a Justify, as does the phrase “allow the conclusion to be properly
drawn.”
“Which one of the following, if true, does most to justify the apparently contradictory
conclusion above?”
Question Type: Strengthen
Stem drawn from the December 2002 LSAT. The presence of the phrase “does most to justify” indicates
that this question stem is a Strengthen. The presence of “most” opens the door to an answer that does not
100% justify the conclusion, hence the question is not a true Justify the Conclusion question.
“Which one of the following, if true, would most effectively resolve the
apparent paradox above?”
Resolve the Paradox
No conclusion, Language of contradiction
The correct answer will actively resolve the paradox—it will allow both sides to
be factually correct and it will either explain how the situation came into being
or add a piece of information that shows how the two ideas or occurrences can
coexist.

Family #2
1) The information in the stimulus is suspect. There are often reasoning errors present, and depending on the question, you will help shore up the argument in some way.

2) The answer choices are accepted as given, even if they include "new" information. Your task is to determine which answer choice best meets the question posed in the stem.

Stimulus (under suspicion) <- Answer Choices
“Which one of the following, if true, most helps to resolve the apparent
discrepancy in the passage above?”
Resolve the Paradox
No conclusion, Language of contradiction
The correct answer will actively resolve the paradox—it will allow both sides to
be factually correct and it will either explain how the situation came into being
or add a piece of information that shows how the two ideas or occurrences can
coexist.

Family #2
1) The information in the stimulus is suspect. There are often reasoning errors present, and depending on the question, you will help shore up the argument in some way.

2) The answer choices are accepted as given, even if they include "new" information. Your task is to determine which answer choice best meets the question posed in the stem.

Stimulus (under suspicion) <- Answer Choices
“Which one of the following, if true, most helps to explain the puzzling
fact cited above?”
Resolve the Paradox
No conclusion, Language of contradiction
The correct answer will actively resolve the paradox—it will allow both sides to
be factually correct and it will either explain how the situation came into being
or add a piece of information that shows how the two ideas or occurrences can
coexist.

Family #2
1) The information in the stimulus is suspect. There are often reasoning errors present, and depending on the question, you will help shore up the argument in some way.

2) The answer choices are accepted as given, even if they include "new" information. Your task is to determine which answer choice best meets the question posed in the stem.

Stimulus (under suspicion) <- Answer Choices
“Which one of the following, if true, most helps to reconcile the
discrepancy indicated above?”
Resolve the Paradox
No conclusion, Language of contradiction
The correct answer will actively resolve the paradox—it will allow both sides to
be factually correct and it will either explain how the situation came into being
or add a piece of information that shows how the two ideas or occurrences can
coexist.

Family #2
1) The information in the stimulus is suspect. There are often reasoning errors present, and depending on the question, you will help shore up the argument in some way.

2) The answer choices are accepted as given, even if they include "new" information. Your task is to determine which answer choice best meets the question posed in the stem.

Stimulus (under suspicion) <- Answer Choices
“Which one of the following, if true, most helps to resolve the apparent
conflict described above?”
Resolve the Paradox
No conclusion, Language of contradiction
The correct answer will actively resolve the paradox—it will allow both sides to
be factually correct and it will either explain how the situation came into being
or add a piece of information that shows how the two ideas or occurrences can
coexist.

Family #2
1) The information in the stimulus is suspect. There are often reasoning errors present, and depending on the question, you will help shore up the argument in some way.

2) The answer choices are accepted as given, even if they include "new" information. Your task is to determine which answer choice best meets the question posed in the stem.

Stimulus (under suspicion) <- Answer Choices
“The method of the argument is to”
METHOD OF REASONING
describe, in abstract terms, the way in which the author made his or her argument.

Fact Test

Wrong answers:
1) "New" elements
2) Half right, half wrong
3) Exaggerated
4) Opposite
5) Reverse

First Family

1) Accept stimulus information - even if contains an error of reasoning - and use it to prove that one of the answer choices must be true.

2) Any information that does not appear either directly or as a combination of items in the stimulus will be incorrect.

Stimulus -> Answer Choices "under suspicion"
“The argument proceeds by”
METHOD OF REASONING
describe, in abstract terms, the way in which the author made his or her argument.

Fact Test

Wrong answers:
1) "New" elements
2) Half right, half wrong
3) Exaggerated
4) Opposite
5) Reverse

First Family

1) Accept stimulus information - even if contains an error of reasoning - and use it to prove that one of the answer choices must be true.

2) Any information that does not appear either directly or as a combination of items in the stimulus will be incorrect.

Stimulus -> Answer Choices "under suspicion"
“The argument derives its conclusion by”
METHOD OF REASONING
describe, in abstract terms, the way in which the author made his or her argument.

Fact Test

Wrong answers:
1) "New" elements
2) Half right, half wrong
3) Exaggerated
4) Opposite
5) Reverse

First Family

1) Accept stimulus information - even if contains an error of reasoning - and use it to prove that one of the answer choices must be true.

2) Any information that does not appear either directly or as a combination of items in the stimulus will be incorrect.

Stimulus -> Answer Choices "under suspicion"
“Which one of the following describes the technique of reasoning used
above?”
METHOD OF REASONING
describe, in abstract terms, the way in which the author made his or her argument.

Fact Test

Wrong answers:
1) "New" elements
2) Half right, half wrong
3) Exaggerated
4) Opposite
5) Reverse

First Family

1) Accept stimulus information - even if contains an error of reasoning - and use it to prove that one of the answer choices must be true.

2) Any information that does not appear either directly or as a combination of items in the stimulus will be incorrect.

Stimulus -> Answer Choices "under suspicion"
“Which one of the following is an argumentative strategy employed in
the argument?”
METHOD OF REASONING
describe, in abstract terms, the way in which the author made his or her argument.

Fact Test

Wrong answers:
1) "New" elements
2) Half right, half wrong
3) Exaggerated
4) Opposite
5) Reverse

First Family

1) Accept stimulus information - even if contains an error of reasoning - and use it to prove that one of the answer choices must be true.

2) Any information that does not appear either directly or as a combination of items in the stimulus will be incorrect.

Stimulus -> Answer Choices "under suspicion"
“The argument employs which one of the following reasoning
techniques?”
METHOD OF REASONING
describe, in abstract terms, the way in which the author made his or her argument.

Fact Test

Wrong answers:
1) "New" elements
2) Half right, half wrong
3) Exaggerated
4) Opposite
5) Reverse

First Family

1) Accept stimulus information - even if contains an error of reasoning - and use it to prove that one of the answer choices must be true.

2) Any information that does not appear either directly or as a combination of items in the stimulus will be incorrect.

Stimulus -> Answer Choices "under suspicion"
“Aiesha responds to Adam’s argument by”
METHOD OF REASONING
describe, in abstract terms, the way in which the author made his or her argument.

Fact Test

Wrong answers:
1) "New" elements
2) Half right, half wrong
3) Exaggerated
4) Opposite
5) Reverse

First Family

1) Accept stimulus information - even if contains an error of reasoning - and use it to prove that one of the answer choices must be true.

2) Any information that does not appear either directly or as a combination of items in the stimulus will be incorrect.

Stimulus -> Answer Choices "under suspicion"
“The claim that inventors sometimes serve as their own engineers plays
which one of the following roles in the argument?”
METHOD OF REASONING - ARGUMENT PARTS

often feature two conclusions—a main conclusion and
subsidiary conclusion—where the main conclusion is typically placed in the
first or second sentence, and the last sentence contains the subsidiary
conclusion. In addition, the subsidiary conclusion is often preceded by a
conclusion indicator such as “thus” or “therefore” while the main conclusion is
not prefaced by an indicator.
“The statement ‘thinking machines closely modeled on the brain are also
likely to fail’ serves which one of the following roles in Yang’s
argument?”
METHOD OF REASONING - ARGUMENT PARTS

often feature two conclusions—a main conclusion and
subsidiary conclusion—where the main conclusion is typically placed in the
first or second sentence, and the last sentence contains the subsidiary
conclusion. In addition, the subsidiary conclusion is often preceded by a
conclusion indicator such as “thus” or “therefore” while the main conclusion is
not prefaced by an indicator.
“The assertion that a later artist tampered with Veronese’s painting
serves which one of the following functions in the curator’s argument?”
METHOD OF REASONING - ARGUMENT PARTS

often feature two conclusions—a main conclusion and
subsidiary conclusion—where the main conclusion is typically placed in the
first or second sentence, and the last sentence contains the subsidiary
conclusion. In addition, the subsidiary conclusion is often preceded by a
conclusion indicator such as “thus” or “therefore” while the main conclusion is
not prefaced by an indicator.
“Which one of the following most accurately describes a flaw in the
argument’s reasoning?”
Flaw in Reasoning- describe, in abstract terms, the error of reasoning committed by the author.

First Family

1) Accept stimulus information - even if contains an error of reasoning - and use it to prove that one of the answer choices must be true.

2) Any information that does not appear either directly or as a combination of items in the stimulus will be incorrect.

Stimulus -> Answer Choices "under suspicion"
“The reasoning in the argument is most vulnerable to criticism on the
ground that the argument”
Flaw in Reasoning- describe, in abstract terms, the error of reasoning committed by the author.

First Family

1) Accept stimulus information - even if contains an error of reasoning - and use it to prove that one of the answer choices must be true.

2) Any information that does not appear either directly or as a combination of items in the stimulus will be incorrect.

Stimulus -> Answer Choices "under suspicion"
“The reasoning above is flawed because it fails to recognize that”
Flaw in Reasoning- describe, in abstract terms, the error of reasoning committed by the author.

First Family

1) Accept stimulus information - even if contains an error of reasoning - and use it to prove that one of the answer choices must be true.

2) Any information that does not appear either directly or as a combination of items in the stimulus will be incorrect.

Stimulus -> Answer Choices "under suspicion"
“A questionable aspect of the reasoning above is that it”
Flaw in Reasoning- describe, in abstract terms, the error of reasoning committed by the author.

First Family

1) Accept stimulus information - even if contains an error of reasoning - and use it to prove that one of the answer choices must be true.

2) Any information that does not appear either directly or as a combination of items in the stimulus will be incorrect.

Stimulus -> Answer Choices "under suspicion"
“The reasoning in the argument is fallacious because the argument”
Flaw in Reasoning- describe, in abstract terms, the error of reasoning committed by the author.

First Family

1) Accept stimulus information - even if contains an error of reasoning - and use it to prove that one of the answer choices must be true.

2) Any information that does not appear either directly or as a combination of items in the stimulus will be incorrect.

Stimulus -> Answer Choices "under suspicion"
“Which one of the following is most closely parallel in its reasoning to
the reasoning in the argument above?”
Parallel Reasoning- identify the asnwer choice that contains reasoning most similar in structure to the reasoning presented in the stimulus.

Must parallel all of the following:
1) Method of Reasoning
2) The Conclusion (use same language)
3) The Premises ( look for same some, most, all language)
4) Validity of the Argument

First Family

1) Accept stimulus information - even if contains an error of reasoning - and use it to prove that one of the answer choices must be true.

2) Any information that does not appear either directly or as a combination of items in the stimulus will be incorrect.

Stimulus -> Answer Choices "under suspicion"
“Which one of the following exhibits a pattern of reasoning most similar
to that exhibited by the argument above?”
Parallel Reasoning- identify the asnwer choice that contains reasoning most similar in structure to the reasoning presented in the stimulus.

Must parallel all of the following:
1) Method of Reasoning
2) The Conclusion (use same language)
3) The Premises ( look for same some, most, all language)
4) Validity of the Argument

First Family

1) Accept stimulus information - even if contains an error of reasoning - and use it to prove that one of the answer choices must be true.

2) Any information that does not appear either directly or as a combination of items in the stimulus will be incorrect.

Stimulus -> Answer Choices "under suspicion"
“Which one of the following arguments is most similar in its logical
features to the argument above?”
Parallel Reasoning- identify the asnwer choice that contains reasoning most similar in structure to the reasoning presented in the stimulus.

Must parallel all of the following:
1) Method of Reasoning
2) The Conclusion (use same language)
3) The Premises ( look for same some, most, all language)
4) Validity of the Argument

First Family

1) Accept stimulus information - even if contains an error of reasoning - and use it to prove that one of the answer choices must be true.

2) Any information that does not appear either directly or as a combination of items in the stimulus will be incorrect.

Stimulus -> Answer Choices "under suspicion"
“Which one of the following arguments is most similar in its pattern of
reasoning to the argument above?”
Parallel Reasoning- identify the asnwer choice that contains reasoning most similar in structure to the reasoning presented in the stimulus.

Must parallel all of the following:
1) Method of Reasoning
2) The Conclusion (use same language)
3) The Premises ( look for same some, most, all language)
4) Validity of the Argument

First Family

1) Accept stimulus information - even if contains an error of reasoning - and use it to prove that one of the answer choices must be true.

2) Any information that does not appear either directly or as a combination of items in the stimulus will be incorrect.

Stimulus -> Answer Choices "under suspicion"
“The structure of the reasoning in the argument above is most parallel to
that in which one of the following?”
Parallel Reasoning- identify the asnwer choice that contains reasoning most similar in structure to the reasoning presented in the stimulus.

Must parallel all of the following:
1) Method of Reasoning
2) The Conclusion (use same language)
3) The Premises ( look for same some, most, all language)
4) Validity of the Argument

First Family

1) Accept stimulus information - even if contains an error of reasoning - and use it to prove that one of the answer choices must be true.

2) Any information that does not appear either directly or as a combination of items in the stimulus will be incorrect.

Stimulus -> Answer Choices "under suspicion"
“The flawed reasoning in which one of the following is most similar to
the flawed reasoning in the argument above?”
Parallel Flaw
“The questionable pattern of reasoning in the argument is most similar to
that in which one of the following?”
Parallel Flaw
MBT
If the stimulus contains percentage or proportion information only, avoid
answers that contain hard numbers.
Example Stimulus Sentence:
The car market share of Company X declined this year.
Avoid answers which say:
Company X sold a smaller number of cars this year.
Company X sold a greater amount of cars this year.
2.
MBT
If the stimulus contains only numerical information, avoid answers that

contain percentage or proportion information.
Example Stimulus Sentence:
Company Y sold fewer computers this year.
Avoid answers which say:
Company Y now has a lower share of the computer market.
Company Y now possesses a greater proportion of the
computer market.
MBT
If the stimulus contains both percentage and numerical information,
any
answer choice that contains numbers, percentages, or both may be true.
“The answer to which one of the following questions would contribute
most to an evaluation of the argument?”
EVALUATE THE ARGUMENT
you must decide which answer choice will allow you to determine the logical validity of the argument.

Use the varience test - supply two opposite responses to the questions posed in the answer choice and then analyze how the varying responses would affect the conclusion in the stimulus. Only apply to contenders.
“Clarification of which one of the following issues would be most
important to an evaluation of the skeptics’ position?”
EVALUATE THE ARGUMENT
you must decide which answer choice will allow you to determine the logical validity of the argument.

Use the varience test - supply two opposite responses to the questions posed in the answer choice and then analyze how the varying responses would affect the conclusion in the stimulus. Only apply to contenders.
“Which one of the following would be most important to know in
evaluating the hypothesis in the passage?”
EVALUATE THE ARGUMENT
you must decide which answer choice will allow you to determine the logical validity of the argument.

Use the varience test - supply two opposite responses to the questions posed in the answer choice and then analyze how the varying responses would affect the conclusion in the stimulus. Only apply to contenders.
“Which one of the following would it be most relevant to investigate in
evaluating the conclusion of George’s argument?”
EVALUATE THE ARGUMENT
you must decide which answer choice will allow you to determine the logical validity of the argument.

Use the varience test - supply two opposite responses to the questions posed in the answer choice and then analyze how the varying responses would affect the conclusion in the stimulus. Only apply to contenders.
“Which one of the following would it be most helpful to know in order
to judge whether what the scientist subsequently learned calls into
question the hypothesis?”
EVALUATE THE ARGUMENT
you must decide which answer choice will allow you to determine the logical validity of the argument.

Use the varience test - supply two opposite responses to the questions posed in the answer choice and then analyze how the varying responses would affect the conclusion in the stimulus. Only apply to contenders.
“If the statements above are true, which one of the following
CANNOT be true?”
CANNOT BE TRUE
If a condition statement, this is the right answer:
The sufficient occurs, and the necessary condition does not occur.


ask you to identify the answer choice that cannot be true or is most weakened based on the information in the stimulus.

Family #4

1) You must accept the stimulus information - even if it contains an error in reasoning - and use it to prove that one of the answer choices cannot occur.

2)If an answer choice contains information that does not appear directly in the stimulus or as a result of combination of items in the stimulus, then that answer choice chould be true and it is incorrect. The correct answer will disagree with the stimulus or a consequence of the stimulus.

Stimulus -/-> Answer Choices
Use information in stimulus to find an answer that CANNOT be true.
“The argument can most reasonably be interpreted as an
objection to which one of the following claims?”
CANNOT BE TRUE
If a condition statement, this is the right answer:
The sufficient occurs, and the necessary condition does not occur.


ask you to identify the answer choice that cannot be true or is most weakened based on the information in the stimulus.

Family #4

1) You must accept the stimulus information - even if it contains an error in reasoning - and use it to prove that one of the answer choices cannot occur.

2)If an answer choice contains information that does not appear directly in the stimulus or as a result of combination of items in the stimulus, then that answer choice chould be true and it is incorrect. The correct answer will disagree with the stimulus or a consequence of the stimulus.

Stimulus -/-> Answer Choices
Use information in stimulus to find an answer that CANNOT be true.
“The statements above, if true, most seriously undermine which
one of the following assertions?”
CANNOT BE TRUE
If a condition statement, this is the right answer:
The sufficient occurs, and the necessary condition does not occur.


ask you to identify the answer choice that cannot be true or is most weakened based on the information in the stimulus.

Family #4

1) You must accept the stimulus information - even if it contains an error in reasoning - and use it to prove that one of the answer choices cannot occur.

2)If an answer choice contains information that does not appear directly in the stimulus or as a result of combination of items in the stimulus, then that answer choice chould be true and it is incorrect. The correct answer will disagree with the stimulus or a consequence of the stimulus.

Stimulus -/-> Answer Choices
Use information in stimulus to find an answer that CANNOT be true.
“The information above, if accurate, can best be used as
evidence against which one of the following hypotheses?”
CANNOT BE TRUE
If a condition statement, this is the right answer:
The sufficient occurs, and the necessary condition does not occur.


ask you to identify the answer choice that cannot be true or is most weakened based on the information in the stimulus.

Family #4

1) You must accept the stimulus information - even if it contains an error in reasoning - and use it to prove that one of the answer choices cannot occur.

2)If an answer choice contains information that does not appear directly in the stimulus or as a result of combination of items in the stimulus, then that answer choice chould be true and it is incorrect. The correct answer will disagree with the stimulus or a consequence of the stimulus.

Stimulus -/-> Answer Choices
Use information in stimulus to find an answer that CANNOT be true.
“If all of the claims made above are true, then each of the
following could be true EXCEPT: ”
CANNOT BE TRUE
If a condition statement, this is the right answer:
The sufficient occurs, and the necessary condition does not occur.


ask you to identify the answer choice that cannot be true or is most weakened based on the information in the stimulus.

Family #4

1) You must accept the stimulus information - even if it contains an error in reasoning - and use it to prove that one of the answer choices cannot occur.

2)If an answer choice contains information that does not appear directly in the stimulus or as a result of combination of items in the stimulus, then that answer choice chould be true and it is incorrect. The correct answer will disagree with the stimulus or a consequence of the stimulus.

Stimulus -/-> Answer Choices
Use information in stimulus to find an answer that CANNOT be true.
“If the statements above are true, then which one of the
following must be false?”
CANNOT BE TRUE
If a condition statement, this is the right answer:
The sufficient occurs, and the necessary condition does not occur.


ask you to identify the answer choice that cannot be true or is most weakened based on the information in the stimulus.

Family #4

1) You must accept the stimulus information - even if it contains an error in reasoning - and use it to prove that one of the answer choices cannot occur.

2)If an answer choice contains information that does not appear directly in the stimulus or as a result of combination of items in the stimulus, then that answer choice chould be true and it is incorrect. The correct answer will disagree with the stimulus or a consequence of the stimulus.

Stimulus -/-> Answer Choices
Use information in stimulus to find an answer that CANNOT be true.
“Which one of the following most accurately expresses the point at issue
between Tom and Mary?”
POINT AT ISSUE
identify a point of contention between two speakers, almost exclusive to two-speaker stimuli.

Incorrect answers:
1) Ethical vs. Factual situations
2) Dual Agreement or Dual Disagreement
3) The view of one speaker is unknown.

Right answer:
Agree/Disagree test: One speaker would agree the statement is correct, the other would disagree with the statement.

First Family

1) Accept stimulus information - even if contains an error of reasoning - and use it to prove that one of the answer choices must be true.

2) Any information that does not appear either directly or as a combination of items in the stimulus will be incorrect.

Stimulus -> Answer Choices "under suspicion"
“Which one of the following most accurately represents what is at issue
between Jorge and Ruth?”
POINT AT ISSUE
identify a point of contention between two speakers, almost exclusive to two-speaker stimuli.

Incorrect answers:
1) Ethical vs. Factual situations
2) Dual Agreement or Dual Disagreement
3) The view of one speaker is unknown.

Right answer:
Agree/Disagree test: One speaker would agree the statement is correct, the other would disagree with the statement.

First Family

1) Accept stimulus information - even if contains an error of reasoning - and use it to prove that one of the answer choices must be true.

2) Any information that does not appear either directly or as a combination of items in the stimulus will be incorrect.

Stimulus -> Answer Choices "under suspicion"
“The dialogue above lends the most support to the claim that Sherrie
and Fran disagree with each other about which one of the following
statements?”
POINT AT ISSUE
identify a point of contention between two speakers, almost exclusive to two-speaker stimuli.

Incorrect answers:
1) Ethical vs. Factual situations
2) Dual Agreement or Dual Disagreement
3) The view of one speaker is unknown.

Right answer:
Agree/Disagree test: One speaker would agree the statement is correct, the other would disagree with the statement.

First Family

1) Accept stimulus information - even if contains an error of reasoning - and use it to prove that one of the answer choices must be true.

2) Any information that does not appear either directly or as a combination of items in the stimulus will be incorrect.

Stimulus -> Answer Choices "under suspicion"
“On the basis of their statement, Logan and Mendez are committed to
disagreeing over whether”
POINT AT ISSUE
identify a point of contention between two speakers, almost exclusive to two-speaker stimuli.

Incorrect answers:
1) Ethical vs. Factual situations
2) Dual Agreement or Dual Disagreement
3) The view of one speaker is unknown.

Right answer:
Agree/Disagree test: One speaker would agree the statement is correct, the other would disagree with the statement.

First Family

1) Accept stimulus information - even if contains an error of reasoning - and use it to prove that one of the answer choices must be true.

2) Any information that does not appear either directly or as a combination of items in the stimulus will be incorrect.

Stimulus -> Answer Choices "under suspicion"
“Which one of the following judgments most closely conforms to the
principle above?”
(Must-PR)
In Must-PR questions you must use the principle presented in the stimulus and
then apply it to the situation in each answer choice (one principle applied to five
situations). The presence of the principle designation broadens the question, and
the answer choice can address a scenario not included in the stimulus.
“Which one of the following judgments best illustrates the principle
illustrated by the argument above?”
(Must-PR)
In Must-PR questions you must use the principle presented in the stimulus and
then apply it to the situation in each answer choice (one principle applied to five
situations). The presence of the principle designation broadens the question, and
the answer choice can address a scenario not included in the stimulus.
“The principle above, if established, would justify which one of the
following judgments?”
(Must-PR
In Must-PR questions you must use the principle presented in the stimulus and
then apply it to the situation in each answer choice (one principle applied to five
situations). The presence of the principle designation broadens the question, and
the answer choice can address a scenario not included in the stimulus.
“Which one of the following principles most helps to justify the
reasoning above?”
(Strengthen-PR)
each answer choice contains a principle that acts as
an additional, broad premise that supports or proves the conclusion
(functionally, five different principles are applied to the situation in the
stimulus).
“The information above most closely conforms to which one of the
following principles?”
(Strengthen-PR)
each answer choice contains a principle that acts as
an additional, broad premise that supports or proves the conclusion
(functionally, five different principles are applied to the situation in the
stimulus).
“Which one of the following most accurately expresses the principle
underlying the argumentation above?”
(Justify-PR)
“Each of the following principles is logically consistent with the
columnist’s conclusion EXCEPT:”
(Cannot-PR)
“Each of the following, if true, supports the claim above
EXCEPT:”
Question Type: StrengthenX
Stem drawn from the October 2003 LSAT. The four incorrect answer choices
Strengthen the argument; the correct answer choice does not Strengthen the
argument.
“Each of the following, if true, weakens the argument
EXCEPT:”
Question Type: WeakenX
Stem drawn from the December 2002 LSAT. The four incorrect answer
choices Weaken the argument; the correct answer choice does not Weaken the
argument.
“Which one of the following, if all of them are true, is
LEAST helpful in establishing that the conclusion above is properly
drawn?”
Question Type: StrengthenX
Stem drawn from the December 2003 LSAT. The four incorrect answer
choices Strengthen the argument (“helpful in establishing the conclusion” is
the same as Strengthen); the correct answer choice does not Strengthen the
argument. The “LEAST” in the stem functions in the same fashion as
“EXCEPT.”
“Each of the following describes a flaw in the
psychologist’s reasoning EXCEPT:”
Question Type: FlawX
Stem drawn from the October 2000 LSAT. The four incorrect answer choices
describe a Flaw in the Reasoning; the correct answer choice does not describe
a Flaw in the Reasoning.
“Which one of the following, if true, does NOT help to
resolve the apparent discrepancy between the safety report and the city’s
public safety record?”
Question Type: ResolveX
Stem drawn from the June 2002 LSAT. Although this question stem uses
neither “except” nor “least,” the use of the word “NOT” indicates that the four
incorrect answer choices Resolve the Paradox and the correct answer choice
does not Resolve the Paradox. Hence, this question is classified ResolveX.
“If the statements above are true, each of the following
could be true EXCEPT:”
Question Type: Cannot Be True
Stem drawn from the October 2003 LSAT. As noted earlier, when the “could
be true EXCEPT” construction appears, the question will be classified as
Cannot Be True. This is because the four incorrect answers are Could Be True,
and the remaining answer choice is the opposite—Cannot Be True.